A magician places $n$ coins, someone choose a coin, for what $n$ the magician will succeed?












8












$begingroup$


There is a riddle which I'm having a hard time to solve, would love some help...



A magician places $n$ coins on a table and walks down off the stage. A volunteer comes, turn over whichever coins he wishes, select one coin and whispers its number to the apprentice. Then the apprentice turns over one coin aiming to assist the magician to know the selected coin. For which values of $n$ the magic will always succeed? what if the apprentice can choose to flip one or zero coins? (instead of always one).










share|cite|improve this question











$endgroup$












  • $begingroup$
    Is the "selected coin" the one the volunteer turns over, or does he turn over one coin and whisper a perhaps different coin in the apprentice's ear?
    $endgroup$
    – saulspatz
    16 hours ago










  • $begingroup$
    @saulspatz After reading again I think one of those he already flipped.
    $endgroup$
    – user2323232
    15 hours ago










  • $begingroup$
    After looking at Empy2's answer, and the comments, I realize I was solving the wrong riddle.
    $endgroup$
    – saulspatz
    14 hours ago










  • $begingroup$
    if the apprentice can choose to flip $0$ coins, then the $n=3$ case can be solved. expanding on the answer by @Empy2 : we have a cube. simply color $3$ pairs of opposite corners with $3$ different colors (and leave the $4$th pair uncolored). any corner is $0$ or $1$ away from any desired color.
    $endgroup$
    – antkam
    13 hours ago










  • $begingroup$
    Does the flipping of a number of coins matter at all? I don't see how it does. As long as the magician knows which coin the assistant flipped it can be done.
    $endgroup$
    – Ross Millikan
    13 hours ago
















8












$begingroup$


There is a riddle which I'm having a hard time to solve, would love some help...



A magician places $n$ coins on a table and walks down off the stage. A volunteer comes, turn over whichever coins he wishes, select one coin and whispers its number to the apprentice. Then the apprentice turns over one coin aiming to assist the magician to know the selected coin. For which values of $n$ the magic will always succeed? what if the apprentice can choose to flip one or zero coins? (instead of always one).










share|cite|improve this question











$endgroup$












  • $begingroup$
    Is the "selected coin" the one the volunteer turns over, or does he turn over one coin and whisper a perhaps different coin in the apprentice's ear?
    $endgroup$
    – saulspatz
    16 hours ago










  • $begingroup$
    @saulspatz After reading again I think one of those he already flipped.
    $endgroup$
    – user2323232
    15 hours ago










  • $begingroup$
    After looking at Empy2's answer, and the comments, I realize I was solving the wrong riddle.
    $endgroup$
    – saulspatz
    14 hours ago










  • $begingroup$
    if the apprentice can choose to flip $0$ coins, then the $n=3$ case can be solved. expanding on the answer by @Empy2 : we have a cube. simply color $3$ pairs of opposite corners with $3$ different colors (and leave the $4$th pair uncolored). any corner is $0$ or $1$ away from any desired color.
    $endgroup$
    – antkam
    13 hours ago










  • $begingroup$
    Does the flipping of a number of coins matter at all? I don't see how it does. As long as the magician knows which coin the assistant flipped it can be done.
    $endgroup$
    – Ross Millikan
    13 hours ago














8












8








8


4



$begingroup$


There is a riddle which I'm having a hard time to solve, would love some help...



A magician places $n$ coins on a table and walks down off the stage. A volunteer comes, turn over whichever coins he wishes, select one coin and whispers its number to the apprentice. Then the apprentice turns over one coin aiming to assist the magician to know the selected coin. For which values of $n$ the magic will always succeed? what if the apprentice can choose to flip one or zero coins? (instead of always one).










share|cite|improve this question











$endgroup$




There is a riddle which I'm having a hard time to solve, would love some help...



A magician places $n$ coins on a table and walks down off the stage. A volunteer comes, turn over whichever coins he wishes, select one coin and whispers its number to the apprentice. Then the apprentice turns over one coin aiming to assist the magician to know the selected coin. For which values of $n$ the magic will always succeed? what if the apprentice can choose to flip one or zero coins? (instead of always one).







probability contest-math puzzle






share|cite|improve this question















share|cite|improve this question













share|cite|improve this question




share|cite|improve this question








edited 12 hours ago









YuiTo Cheng

2,4064937




2,4064937










asked 16 hours ago









user2323232user2323232

18810




18810












  • $begingroup$
    Is the "selected coin" the one the volunteer turns over, or does he turn over one coin and whisper a perhaps different coin in the apprentice's ear?
    $endgroup$
    – saulspatz
    16 hours ago










  • $begingroup$
    @saulspatz After reading again I think one of those he already flipped.
    $endgroup$
    – user2323232
    15 hours ago










  • $begingroup$
    After looking at Empy2's answer, and the comments, I realize I was solving the wrong riddle.
    $endgroup$
    – saulspatz
    14 hours ago










  • $begingroup$
    if the apprentice can choose to flip $0$ coins, then the $n=3$ case can be solved. expanding on the answer by @Empy2 : we have a cube. simply color $3$ pairs of opposite corners with $3$ different colors (and leave the $4$th pair uncolored). any corner is $0$ or $1$ away from any desired color.
    $endgroup$
    – antkam
    13 hours ago










  • $begingroup$
    Does the flipping of a number of coins matter at all? I don't see how it does. As long as the magician knows which coin the assistant flipped it can be done.
    $endgroup$
    – Ross Millikan
    13 hours ago


















  • $begingroup$
    Is the "selected coin" the one the volunteer turns over, or does he turn over one coin and whisper a perhaps different coin in the apprentice's ear?
    $endgroup$
    – saulspatz
    16 hours ago










  • $begingroup$
    @saulspatz After reading again I think one of those he already flipped.
    $endgroup$
    – user2323232
    15 hours ago










  • $begingroup$
    After looking at Empy2's answer, and the comments, I realize I was solving the wrong riddle.
    $endgroup$
    – saulspatz
    14 hours ago










  • $begingroup$
    if the apprentice can choose to flip $0$ coins, then the $n=3$ case can be solved. expanding on the answer by @Empy2 : we have a cube. simply color $3$ pairs of opposite corners with $3$ different colors (and leave the $4$th pair uncolored). any corner is $0$ or $1$ away from any desired color.
    $endgroup$
    – antkam
    13 hours ago










  • $begingroup$
    Does the flipping of a number of coins matter at all? I don't see how it does. As long as the magician knows which coin the assistant flipped it can be done.
    $endgroup$
    – Ross Millikan
    13 hours ago
















$begingroup$
Is the "selected coin" the one the volunteer turns over, or does he turn over one coin and whisper a perhaps different coin in the apprentice's ear?
$endgroup$
– saulspatz
16 hours ago




$begingroup$
Is the "selected coin" the one the volunteer turns over, or does he turn over one coin and whisper a perhaps different coin in the apprentice's ear?
$endgroup$
– saulspatz
16 hours ago












$begingroup$
@saulspatz After reading again I think one of those he already flipped.
$endgroup$
– user2323232
15 hours ago




$begingroup$
@saulspatz After reading again I think one of those he already flipped.
$endgroup$
– user2323232
15 hours ago












$begingroup$
After looking at Empy2's answer, and the comments, I realize I was solving the wrong riddle.
$endgroup$
– saulspatz
14 hours ago




$begingroup$
After looking at Empy2's answer, and the comments, I realize I was solving the wrong riddle.
$endgroup$
– saulspatz
14 hours ago












$begingroup$
if the apprentice can choose to flip $0$ coins, then the $n=3$ case can be solved. expanding on the answer by @Empy2 : we have a cube. simply color $3$ pairs of opposite corners with $3$ different colors (and leave the $4$th pair uncolored). any corner is $0$ or $1$ away from any desired color.
$endgroup$
– antkam
13 hours ago




$begingroup$
if the apprentice can choose to flip $0$ coins, then the $n=3$ case can be solved. expanding on the answer by @Empy2 : we have a cube. simply color $3$ pairs of opposite corners with $3$ different colors (and leave the $4$th pair uncolored). any corner is $0$ or $1$ away from any desired color.
$endgroup$
– antkam
13 hours ago












$begingroup$
Does the flipping of a number of coins matter at all? I don't see how it does. As long as the magician knows which coin the assistant flipped it can be done.
$endgroup$
– Ross Millikan
13 hours ago




$begingroup$
Does the flipping of a number of coins matter at all? I don't see how it does. As long as the magician knows which coin the assistant flipped it can be done.
$endgroup$
– Ross Millikan
13 hours ago










3 Answers
3






active

oldest

votes


















5












$begingroup$

I think a better way to represent the $2^n$ strategy in Empty2's answer is to number the coins in binary with $n$ bits. The selected coin has a particular number. The assistant is to make the XOR of all the heads match the selected coin. It can be done by taking the bitwise XOR of the current state with the selected coin and flipping the coin with that number.



As an example, with eight coins suppose coins $1,3,6$ are heads with XOR of $4$. If the selected coin is $5$, flipping the $1$ coin makes the XOR be $5$. If the selected coin is $2$, flipping the $6$ makes the $XOR$ be $2$. If the selected coin is $6$, flipping the $2$ coin gives the proper $XOR$. Flipping the $0$ coin is free, so the assistant can always flip a coin.






share|cite|improve this answer









$endgroup$













  • $begingroup$
    Nice approach. +1.
    $endgroup$
    – Empy2
    12 hours ago










  • $begingroup$
    That solves $n=2^m-1$ coins when the assistant can flip no coins. Just remove coin $0$.
    $endgroup$
    – Empy2
    12 hours ago










  • $begingroup$
    Well, I tried doing an example for n = 4, and your approach doesn't work.
    $endgroup$
    – Ilan Aizelman WS
    7 hours ago



















2












$begingroup$

There are $2^n$ coin positions. Represent them as the vertices of a hypercube. Give each vertex a color to represent one of the $n$ named coins. There must be one of each colour adjacent to any starting position, so there are equal numbers of each colour. So $n$ must be a factor of $2^n$, so $n$ must be a power of 2.



That does't work if the assistant may turn 0 coins, and doesn't explain how to do it for $n=2^m$ coins.






share|cite|improve this answer









$endgroup$













  • $begingroup$
    There are only ${nchoose2}$ allowable positions that are different from the one the magician started with.
    $endgroup$
    – saulspatz
    14 hours ago










  • $begingroup$
    I think that 'turns over whichever coins he wishes' means that all positions are allowable
    $endgroup$
    – Empy2
    14 hours ago






  • 1




    $begingroup$
    I think you're right. I misread it as "coin" instead of "coins".
    $endgroup$
    – saulspatz
    14 hours ago






  • 2




    $begingroup$
    @antkam For your sake, I hope your mind is greater than mine.
    $endgroup$
    – saulspatz
    13 hours ago






  • 1




    $begingroup$
    @saulspatz -- lets just say, by solving the $n=3$ case, we both achieved the same (not very high) lower bound for the greatness of our minds
    $endgroup$
    – antkam
    13 hours ago



















0












$begingroup$

Nevermind. I overlooked the fact that once some coins are flipped, then not all changes to the sum (mod n) are available to the assistant with a single flip.




Number the coins from 1 to n. Let S equal 1 plus (the sum of numbers on the flipped coins modulo n). A set of flipped coins "indicates" the chosen coin which matches S. The assistant can flip any 1 coin, and the coins have each possible value modulo n, so the assistant can indicate which coin was selected by flipping the coin corresponding to the difference between the current S and the desired S.







share|cite|improve this answer










New contributor




Mike H is a new contributor to this site. Take care in asking for clarification, commenting, and answering.
Check out our Code of Conduct.






$endgroup$









  • 1




    $begingroup$
    This doesn't seem to work, at least not without some restriction on $n$. Suppose $n=3,$ the selected coin is $3$, and coins $1$ and $3$ are flipped. The only ways the coins can indicate $3$ are for only coin $2$ to be flipped, or for coins $2$ and $3$ to be flipped, and neither of these is reachable from the initial state in one flip.
    $endgroup$
    – saulspatz
    11 hours ago












Your Answer








StackExchange.ready(function() {
var channelOptions = {
tags: "".split(" "),
id: "69"
};
initTagRenderer("".split(" "), "".split(" "), channelOptions);

StackExchange.using("externalEditor", function() {
// Have to fire editor after snippets, if snippets enabled
if (StackExchange.settings.snippets.snippetsEnabled) {
StackExchange.using("snippets", function() {
createEditor();
});
}
else {
createEditor();
}
});

function createEditor() {
StackExchange.prepareEditor({
heartbeatType: 'answer',
autoActivateHeartbeat: false,
convertImagesToLinks: true,
noModals: true,
showLowRepImageUploadWarning: true,
reputationToPostImages: 10,
bindNavPrevention: true,
postfix: "",
imageUploader: {
brandingHtml: "Powered by u003ca class="icon-imgur-white" href="https://imgur.com/"u003eu003c/au003e",
contentPolicyHtml: "User contributions licensed under u003ca href="https://creativecommons.org/licenses/by-sa/3.0/"u003ecc by-sa 3.0 with attribution requiredu003c/au003e u003ca href="https://stackoverflow.com/legal/content-policy"u003e(content policy)u003c/au003e",
allowUrls: true
},
noCode: true, onDemand: true,
discardSelector: ".discard-answer"
,immediatelyShowMarkdownHelp:true
});


}
});














draft saved

draft discarded


















StackExchange.ready(
function () {
StackExchange.openid.initPostLogin('.new-post-login', 'https%3a%2f%2fmath.stackexchange.com%2fquestions%2f3184968%2fa-magician-places-n-coins-someone-choose-a-coin-for-what-n-the-magician-wi%23new-answer', 'question_page');
}
);

Post as a guest















Required, but never shown

























3 Answers
3






active

oldest

votes








3 Answers
3






active

oldest

votes









active

oldest

votes






active

oldest

votes









5












$begingroup$

I think a better way to represent the $2^n$ strategy in Empty2's answer is to number the coins in binary with $n$ bits. The selected coin has a particular number. The assistant is to make the XOR of all the heads match the selected coin. It can be done by taking the bitwise XOR of the current state with the selected coin and flipping the coin with that number.



As an example, with eight coins suppose coins $1,3,6$ are heads with XOR of $4$. If the selected coin is $5$, flipping the $1$ coin makes the XOR be $5$. If the selected coin is $2$, flipping the $6$ makes the $XOR$ be $2$. If the selected coin is $6$, flipping the $2$ coin gives the proper $XOR$. Flipping the $0$ coin is free, so the assistant can always flip a coin.






share|cite|improve this answer









$endgroup$













  • $begingroup$
    Nice approach. +1.
    $endgroup$
    – Empy2
    12 hours ago










  • $begingroup$
    That solves $n=2^m-1$ coins when the assistant can flip no coins. Just remove coin $0$.
    $endgroup$
    – Empy2
    12 hours ago










  • $begingroup$
    Well, I tried doing an example for n = 4, and your approach doesn't work.
    $endgroup$
    – Ilan Aizelman WS
    7 hours ago
















5












$begingroup$

I think a better way to represent the $2^n$ strategy in Empty2's answer is to number the coins in binary with $n$ bits. The selected coin has a particular number. The assistant is to make the XOR of all the heads match the selected coin. It can be done by taking the bitwise XOR of the current state with the selected coin and flipping the coin with that number.



As an example, with eight coins suppose coins $1,3,6$ are heads with XOR of $4$. If the selected coin is $5$, flipping the $1$ coin makes the XOR be $5$. If the selected coin is $2$, flipping the $6$ makes the $XOR$ be $2$. If the selected coin is $6$, flipping the $2$ coin gives the proper $XOR$. Flipping the $0$ coin is free, so the assistant can always flip a coin.






share|cite|improve this answer









$endgroup$













  • $begingroup$
    Nice approach. +1.
    $endgroup$
    – Empy2
    12 hours ago










  • $begingroup$
    That solves $n=2^m-1$ coins when the assistant can flip no coins. Just remove coin $0$.
    $endgroup$
    – Empy2
    12 hours ago










  • $begingroup$
    Well, I tried doing an example for n = 4, and your approach doesn't work.
    $endgroup$
    – Ilan Aizelman WS
    7 hours ago














5












5








5





$begingroup$

I think a better way to represent the $2^n$ strategy in Empty2's answer is to number the coins in binary with $n$ bits. The selected coin has a particular number. The assistant is to make the XOR of all the heads match the selected coin. It can be done by taking the bitwise XOR of the current state with the selected coin and flipping the coin with that number.



As an example, with eight coins suppose coins $1,3,6$ are heads with XOR of $4$. If the selected coin is $5$, flipping the $1$ coin makes the XOR be $5$. If the selected coin is $2$, flipping the $6$ makes the $XOR$ be $2$. If the selected coin is $6$, flipping the $2$ coin gives the proper $XOR$. Flipping the $0$ coin is free, so the assistant can always flip a coin.






share|cite|improve this answer









$endgroup$



I think a better way to represent the $2^n$ strategy in Empty2's answer is to number the coins in binary with $n$ bits. The selected coin has a particular number. The assistant is to make the XOR of all the heads match the selected coin. It can be done by taking the bitwise XOR of the current state with the selected coin and flipping the coin with that number.



As an example, with eight coins suppose coins $1,3,6$ are heads with XOR of $4$. If the selected coin is $5$, flipping the $1$ coin makes the XOR be $5$. If the selected coin is $2$, flipping the $6$ makes the $XOR$ be $2$. If the selected coin is $6$, flipping the $2$ coin gives the proper $XOR$. Flipping the $0$ coin is free, so the assistant can always flip a coin.







share|cite|improve this answer












share|cite|improve this answer



share|cite|improve this answer










answered 12 hours ago









Ross MillikanRoss Millikan

301k24200375




301k24200375












  • $begingroup$
    Nice approach. +1.
    $endgroup$
    – Empy2
    12 hours ago










  • $begingroup$
    That solves $n=2^m-1$ coins when the assistant can flip no coins. Just remove coin $0$.
    $endgroup$
    – Empy2
    12 hours ago










  • $begingroup$
    Well, I tried doing an example for n = 4, and your approach doesn't work.
    $endgroup$
    – Ilan Aizelman WS
    7 hours ago


















  • $begingroup$
    Nice approach. +1.
    $endgroup$
    – Empy2
    12 hours ago










  • $begingroup$
    That solves $n=2^m-1$ coins when the assistant can flip no coins. Just remove coin $0$.
    $endgroup$
    – Empy2
    12 hours ago










  • $begingroup$
    Well, I tried doing an example for n = 4, and your approach doesn't work.
    $endgroup$
    – Ilan Aizelman WS
    7 hours ago
















$begingroup$
Nice approach. +1.
$endgroup$
– Empy2
12 hours ago




$begingroup$
Nice approach. +1.
$endgroup$
– Empy2
12 hours ago












$begingroup$
That solves $n=2^m-1$ coins when the assistant can flip no coins. Just remove coin $0$.
$endgroup$
– Empy2
12 hours ago




$begingroup$
That solves $n=2^m-1$ coins when the assistant can flip no coins. Just remove coin $0$.
$endgroup$
– Empy2
12 hours ago












$begingroup$
Well, I tried doing an example for n = 4, and your approach doesn't work.
$endgroup$
– Ilan Aizelman WS
7 hours ago




$begingroup$
Well, I tried doing an example for n = 4, and your approach doesn't work.
$endgroup$
– Ilan Aizelman WS
7 hours ago











2












$begingroup$

There are $2^n$ coin positions. Represent them as the vertices of a hypercube. Give each vertex a color to represent one of the $n$ named coins. There must be one of each colour adjacent to any starting position, so there are equal numbers of each colour. So $n$ must be a factor of $2^n$, so $n$ must be a power of 2.



That does't work if the assistant may turn 0 coins, and doesn't explain how to do it for $n=2^m$ coins.






share|cite|improve this answer









$endgroup$













  • $begingroup$
    There are only ${nchoose2}$ allowable positions that are different from the one the magician started with.
    $endgroup$
    – saulspatz
    14 hours ago










  • $begingroup$
    I think that 'turns over whichever coins he wishes' means that all positions are allowable
    $endgroup$
    – Empy2
    14 hours ago






  • 1




    $begingroup$
    I think you're right. I misread it as "coin" instead of "coins".
    $endgroup$
    – saulspatz
    14 hours ago






  • 2




    $begingroup$
    @antkam For your sake, I hope your mind is greater than mine.
    $endgroup$
    – saulspatz
    13 hours ago






  • 1




    $begingroup$
    @saulspatz -- lets just say, by solving the $n=3$ case, we both achieved the same (not very high) lower bound for the greatness of our minds
    $endgroup$
    – antkam
    13 hours ago
















2












$begingroup$

There are $2^n$ coin positions. Represent them as the vertices of a hypercube. Give each vertex a color to represent one of the $n$ named coins. There must be one of each colour adjacent to any starting position, so there are equal numbers of each colour. So $n$ must be a factor of $2^n$, so $n$ must be a power of 2.



That does't work if the assistant may turn 0 coins, and doesn't explain how to do it for $n=2^m$ coins.






share|cite|improve this answer









$endgroup$













  • $begingroup$
    There are only ${nchoose2}$ allowable positions that are different from the one the magician started with.
    $endgroup$
    – saulspatz
    14 hours ago










  • $begingroup$
    I think that 'turns over whichever coins he wishes' means that all positions are allowable
    $endgroup$
    – Empy2
    14 hours ago






  • 1




    $begingroup$
    I think you're right. I misread it as "coin" instead of "coins".
    $endgroup$
    – saulspatz
    14 hours ago






  • 2




    $begingroup$
    @antkam For your sake, I hope your mind is greater than mine.
    $endgroup$
    – saulspatz
    13 hours ago






  • 1




    $begingroup$
    @saulspatz -- lets just say, by solving the $n=3$ case, we both achieved the same (not very high) lower bound for the greatness of our minds
    $endgroup$
    – antkam
    13 hours ago














2












2








2





$begingroup$

There are $2^n$ coin positions. Represent them as the vertices of a hypercube. Give each vertex a color to represent one of the $n$ named coins. There must be one of each colour adjacent to any starting position, so there are equal numbers of each colour. So $n$ must be a factor of $2^n$, so $n$ must be a power of 2.



That does't work if the assistant may turn 0 coins, and doesn't explain how to do it for $n=2^m$ coins.






share|cite|improve this answer









$endgroup$



There are $2^n$ coin positions. Represent them as the vertices of a hypercube. Give each vertex a color to represent one of the $n$ named coins. There must be one of each colour adjacent to any starting position, so there are equal numbers of each colour. So $n$ must be a factor of $2^n$, so $n$ must be a power of 2.



That does't work if the assistant may turn 0 coins, and doesn't explain how to do it for $n=2^m$ coins.







share|cite|improve this answer












share|cite|improve this answer



share|cite|improve this answer










answered 15 hours ago









Empy2Empy2

33.7k12562




33.7k12562












  • $begingroup$
    There are only ${nchoose2}$ allowable positions that are different from the one the magician started with.
    $endgroup$
    – saulspatz
    14 hours ago










  • $begingroup$
    I think that 'turns over whichever coins he wishes' means that all positions are allowable
    $endgroup$
    – Empy2
    14 hours ago






  • 1




    $begingroup$
    I think you're right. I misread it as "coin" instead of "coins".
    $endgroup$
    – saulspatz
    14 hours ago






  • 2




    $begingroup$
    @antkam For your sake, I hope your mind is greater than mine.
    $endgroup$
    – saulspatz
    13 hours ago






  • 1




    $begingroup$
    @saulspatz -- lets just say, by solving the $n=3$ case, we both achieved the same (not very high) lower bound for the greatness of our minds
    $endgroup$
    – antkam
    13 hours ago


















  • $begingroup$
    There are only ${nchoose2}$ allowable positions that are different from the one the magician started with.
    $endgroup$
    – saulspatz
    14 hours ago










  • $begingroup$
    I think that 'turns over whichever coins he wishes' means that all positions are allowable
    $endgroup$
    – Empy2
    14 hours ago






  • 1




    $begingroup$
    I think you're right. I misread it as "coin" instead of "coins".
    $endgroup$
    – saulspatz
    14 hours ago






  • 2




    $begingroup$
    @antkam For your sake, I hope your mind is greater than mine.
    $endgroup$
    – saulspatz
    13 hours ago






  • 1




    $begingroup$
    @saulspatz -- lets just say, by solving the $n=3$ case, we both achieved the same (not very high) lower bound for the greatness of our minds
    $endgroup$
    – antkam
    13 hours ago
















$begingroup$
There are only ${nchoose2}$ allowable positions that are different from the one the magician started with.
$endgroup$
– saulspatz
14 hours ago




$begingroup$
There are only ${nchoose2}$ allowable positions that are different from the one the magician started with.
$endgroup$
– saulspatz
14 hours ago












$begingroup$
I think that 'turns over whichever coins he wishes' means that all positions are allowable
$endgroup$
– Empy2
14 hours ago




$begingroup$
I think that 'turns over whichever coins he wishes' means that all positions are allowable
$endgroup$
– Empy2
14 hours ago




1




1




$begingroup$
I think you're right. I misread it as "coin" instead of "coins".
$endgroup$
– saulspatz
14 hours ago




$begingroup$
I think you're right. I misread it as "coin" instead of "coins".
$endgroup$
– saulspatz
14 hours ago




2




2




$begingroup$
@antkam For your sake, I hope your mind is greater than mine.
$endgroup$
– saulspatz
13 hours ago




$begingroup$
@antkam For your sake, I hope your mind is greater than mine.
$endgroup$
– saulspatz
13 hours ago




1




1




$begingroup$
@saulspatz -- lets just say, by solving the $n=3$ case, we both achieved the same (not very high) lower bound for the greatness of our minds
$endgroup$
– antkam
13 hours ago




$begingroup$
@saulspatz -- lets just say, by solving the $n=3$ case, we both achieved the same (not very high) lower bound for the greatness of our minds
$endgroup$
– antkam
13 hours ago











0












$begingroup$

Nevermind. I overlooked the fact that once some coins are flipped, then not all changes to the sum (mod n) are available to the assistant with a single flip.




Number the coins from 1 to n. Let S equal 1 plus (the sum of numbers on the flipped coins modulo n). A set of flipped coins "indicates" the chosen coin which matches S. The assistant can flip any 1 coin, and the coins have each possible value modulo n, so the assistant can indicate which coin was selected by flipping the coin corresponding to the difference between the current S and the desired S.







share|cite|improve this answer










New contributor




Mike H is a new contributor to this site. Take care in asking for clarification, commenting, and answering.
Check out our Code of Conduct.






$endgroup$









  • 1




    $begingroup$
    This doesn't seem to work, at least not without some restriction on $n$. Suppose $n=3,$ the selected coin is $3$, and coins $1$ and $3$ are flipped. The only ways the coins can indicate $3$ are for only coin $2$ to be flipped, or for coins $2$ and $3$ to be flipped, and neither of these is reachable from the initial state in one flip.
    $endgroup$
    – saulspatz
    11 hours ago
















0












$begingroup$

Nevermind. I overlooked the fact that once some coins are flipped, then not all changes to the sum (mod n) are available to the assistant with a single flip.




Number the coins from 1 to n. Let S equal 1 plus (the sum of numbers on the flipped coins modulo n). A set of flipped coins "indicates" the chosen coin which matches S. The assistant can flip any 1 coin, and the coins have each possible value modulo n, so the assistant can indicate which coin was selected by flipping the coin corresponding to the difference between the current S and the desired S.







share|cite|improve this answer










New contributor




Mike H is a new contributor to this site. Take care in asking for clarification, commenting, and answering.
Check out our Code of Conduct.






$endgroup$









  • 1




    $begingroup$
    This doesn't seem to work, at least not without some restriction on $n$. Suppose $n=3,$ the selected coin is $3$, and coins $1$ and $3$ are flipped. The only ways the coins can indicate $3$ are for only coin $2$ to be flipped, or for coins $2$ and $3$ to be flipped, and neither of these is reachable from the initial state in one flip.
    $endgroup$
    – saulspatz
    11 hours ago














0












0








0





$begingroup$

Nevermind. I overlooked the fact that once some coins are flipped, then not all changes to the sum (mod n) are available to the assistant with a single flip.




Number the coins from 1 to n. Let S equal 1 plus (the sum of numbers on the flipped coins modulo n). A set of flipped coins "indicates" the chosen coin which matches S. The assistant can flip any 1 coin, and the coins have each possible value modulo n, so the assistant can indicate which coin was selected by flipping the coin corresponding to the difference between the current S and the desired S.







share|cite|improve this answer










New contributor




Mike H is a new contributor to this site. Take care in asking for clarification, commenting, and answering.
Check out our Code of Conduct.






$endgroup$



Nevermind. I overlooked the fact that once some coins are flipped, then not all changes to the sum (mod n) are available to the assistant with a single flip.




Number the coins from 1 to n. Let S equal 1 plus (the sum of numbers on the flipped coins modulo n). A set of flipped coins "indicates" the chosen coin which matches S. The assistant can flip any 1 coin, and the coins have each possible value modulo n, so the assistant can indicate which coin was selected by flipping the coin corresponding to the difference between the current S and the desired S.








share|cite|improve this answer










New contributor




Mike H is a new contributor to this site. Take care in asking for clarification, commenting, and answering.
Check out our Code of Conduct.









share|cite|improve this answer



share|cite|improve this answer








edited 10 hours ago





















New contributor




Mike H is a new contributor to this site. Take care in asking for clarification, commenting, and answering.
Check out our Code of Conduct.









answered 11 hours ago









Mike HMike H

11




11




New contributor




Mike H is a new contributor to this site. Take care in asking for clarification, commenting, and answering.
Check out our Code of Conduct.





New contributor





Mike H is a new contributor to this site. Take care in asking for clarification, commenting, and answering.
Check out our Code of Conduct.






Mike H is a new contributor to this site. Take care in asking for clarification, commenting, and answering.
Check out our Code of Conduct.








  • 1




    $begingroup$
    This doesn't seem to work, at least not without some restriction on $n$. Suppose $n=3,$ the selected coin is $3$, and coins $1$ and $3$ are flipped. The only ways the coins can indicate $3$ are for only coin $2$ to be flipped, or for coins $2$ and $3$ to be flipped, and neither of these is reachable from the initial state in one flip.
    $endgroup$
    – saulspatz
    11 hours ago














  • 1




    $begingroup$
    This doesn't seem to work, at least not without some restriction on $n$. Suppose $n=3,$ the selected coin is $3$, and coins $1$ and $3$ are flipped. The only ways the coins can indicate $3$ are for only coin $2$ to be flipped, or for coins $2$ and $3$ to be flipped, and neither of these is reachable from the initial state in one flip.
    $endgroup$
    – saulspatz
    11 hours ago








1




1




$begingroup$
This doesn't seem to work, at least not without some restriction on $n$. Suppose $n=3,$ the selected coin is $3$, and coins $1$ and $3$ are flipped. The only ways the coins can indicate $3$ are for only coin $2$ to be flipped, or for coins $2$ and $3$ to be flipped, and neither of these is reachable from the initial state in one flip.
$endgroup$
– saulspatz
11 hours ago




$begingroup$
This doesn't seem to work, at least not without some restriction on $n$. Suppose $n=3,$ the selected coin is $3$, and coins $1$ and $3$ are flipped. The only ways the coins can indicate $3$ are for only coin $2$ to be flipped, or for coins $2$ and $3$ to be flipped, and neither of these is reachable from the initial state in one flip.
$endgroup$
– saulspatz
11 hours ago


















draft saved

draft discarded




















































Thanks for contributing an answer to Mathematics Stack Exchange!


  • Please be sure to answer the question. Provide details and share your research!

But avoid



  • Asking for help, clarification, or responding to other answers.

  • Making statements based on opinion; back them up with references or personal experience.


Use MathJax to format equations. MathJax reference.


To learn more, see our tips on writing great answers.




draft saved


draft discarded














StackExchange.ready(
function () {
StackExchange.openid.initPostLogin('.new-post-login', 'https%3a%2f%2fmath.stackexchange.com%2fquestions%2f3184968%2fa-magician-places-n-coins-someone-choose-a-coin-for-what-n-the-magician-wi%23new-answer', 'question_page');
}
);

Post as a guest















Required, but never shown





















































Required, but never shown














Required, but never shown












Required, but never shown







Required, but never shown

































Required, but never shown














Required, but never shown












Required, but never shown







Required, but never shown







Popular posts from this blog

Statuo de Libereco

Tanganjiko

Liste der Baudenkmäler in Enneberg